Integrating Gaussian functions with erf

Click For Summary
The discussion focuses on integrating Gaussian functions, specifically the integrals of e^{-x^2}, x e^{-x^2}, and x^2 e^{-x^2}. The error function (erf) is utilized to simplify the first integral, while integration by parts is applied to the second integral. For the third integral, the user seeks a simpler method than integration by parts, as the textbook provides a concise solution. The user expresses frustration over the complexity of the problem compared to the straightforward solution in the book. The thread highlights the challenge of integrating higher powers of x in Gaussian functions and the desire for more efficient techniques.
bigevil
Messages
77
Reaction score
0

Homework Statement



I'm doing a problem on Gaussian functions (there are other constants to make it interesting, but I've removed them here):

1. \int_{0}^{x} e^{-x^2} dx
2. \int_{0}^{x} x e^{-x^2} dx
3. \int_{0}^{x} x^2 e^{-x^2} dx



We know that

erf(z) = \frac{2}{\sqrt{\pi}} \int_{0}^{x} e^{-x^2} dx

I can use the error function to solve 1. For 2, I integrate by parts:

\int_{0}^{x} x e^{-x^2} dx = \big[ \frac{\sqrt{\pi}}{2} x.erf(x) \big]_0^{x} - \int_{0}^{x} e^{-x^2} dx =\frac{\sqrt{\pi}}{2} x . erf(x) - \frac{\sqrt{\pi}}{2} . erf(x)

Now I want to integrate part 3, and an obvious route to go is integration by parts, but is there a simpler way? Surely I'm not going to do IBP if I was given an integral of an even higher power? I'm doing a question in a (physics) textbook with all the constants and that to make it interesting, but it really boils down to this. My book gives a one line working, which is as follows:

\int_{0}^{32} x^2 e^{-x^{2}/4} = - \frac{64}{256} + 2 \sqrt{\pi} erf[16]

Homework Equations



Is there a simple, obvious way to do this integral without IBP? I do know that

\int_{-\infty}^{\infty} x^{2n} e^{-x^2} dz = \sqrt{\pi} \frac{1.3.5...(2n-1)}{2^n}


The Attempt at a Solution



Well, I tried doing the integration by parts, which was fine, but seeing as my book provides a one-line working for this integral, I figure it should be even easier than that.
 
Last edited:
Physics news on Phys.org
Hey, what happened to my Tex??

Test:
\leftrightarrow\rightarrow\rightharpoonup\rightharpoondown
 
Question: A clock's minute hand has length 4 and its hour hand has length 3. What is the distance between the tips at the moment when it is increasing most rapidly?(Putnam Exam Question) Answer: Making assumption that both the hands moves at constant angular velocities, the answer is ## \sqrt{7} .## But don't you think this assumption is somewhat doubtful and wrong?

Similar threads

Replies
4
Views
2K
  • · Replies 7 ·
Replies
7
Views
2K
  • · Replies 2 ·
Replies
2
Views
2K
Replies
1
Views
2K
  • · Replies 14 ·
Replies
14
Views
2K
  • · Replies 6 ·
Replies
6
Views
2K
  • · Replies 19 ·
Replies
19
Views
2K
  • · Replies 5 ·
Replies
5
Views
2K
  • · Replies 5 ·
Replies
5
Views
2K
  • · Replies 9 ·
Replies
9
Views
3K